Moment of inertia of an isosceles triangle

In summary: The moment of inertia of a thin strip about its midpoint is a well known formula. That plus the parallel axis theorem gets you to the moment of inertia of a thin strip about the apex...
  • #1
Nexus99
103
9
Homework Statement
Find the moment of inertia of an isosceles triangle of mass M = 1.0 kg, height h = 0.4 m and base angles equal to ## \alpha = \frac{ \pi}{6} ##, with respect to an axis passing through its vertex
Relevant Equations
moment of inertia
Cattura.PNG

I did in this way:
## I = \int dm \rho^2 ##
Dividing the triangle in small rectangles with ##dA = dy x(y) ## where ##x(y) = 2 ctg( \alpha ) (h - y) ##
we have : ## dm = \sigma 2 ctg( \alpha ) (h - y) ##
Now i have ## \rho^2 = x^2 + (h-y)^2 ##
Now I don't know what I can do because it would be an integral in 2 variables that I don't know how to do
 
Physics news on Phys.org
  • #2
Why not just set up an integral in ##dxdy##?
 
  • #3
PeroK said:
Why not just set up an integral in ##dxdy##?
I can't do for the moment integrals with two variables
 
  • #4
Nexus99 said:
I can't do for the moment integrals with two variables
That might be a problem. I'm not sure how you could avoid a double integral.
 
  • #5
As has been mentioned, it's easier to impose a Cartesian frame (e.g. origin at top of triangle, ##x## axis running along the height), then the region of integration is specified by ##-\frac{x}{\tan{\alpha}} \leq y \leq \frac{x}{\tan{\alpha}}## and ##0 \leq x \leq h##, so you can write down the integral$$I_z = \int_S \rho r^2 dx dy = \int_0^h \int_{-\frac{x}{\tan{\alpha}}}^{\frac{x}{\tan{\alpha}}} \rho (x^2 + y^2) dy dx$$where ##\rho## is the area density. To evaluate this you can do the inner integral first, holding ##x## constant, and then feed the result into the outer integral.
 
  • Like
Likes Nexus99 and PeroK
  • #6
etotheipi said:
As has been mentioned, it's easier to impose a Cartesian frame (e.g. origin at top of triangle, ##x## axis running along the height), then the region of integration is specified by ##-\frac{x}{\tan{\alpha}} \leq y \leq \frac{x}{\tan{\alpha}}## and ##0 \leq x \leq h##, so you can write down the integral$$I_z = \int_S \rho r^2 dx dy = \int_0^h \int_{-\frac{x}{\tan{\alpha}}}^{\frac{x}{\tan{\alpha}}} \rho (x^2 + y^2) dy dx$$where ##\rho## is the area density. To evaluate this you can do the inner integral first, holding ##x## constant, and then feed the result into the outer integral.
I am sure that is easier, but i have never done a multivariable integral.
Is it possible tu use the fact that:
## I_z = I_x + I_y ##?
 
  • #7
Nexus99 said:
I am sure that is easier, but i have never done a multivariable integral.
Is it possible tu use the fact that:
## I_z = I_x + I_y ##?

That is indeed true for a lamina, it follows because of the linearity of the integral$$I_z = \int_S \rho r^2 dS = \int_S \rho (x^2 + y^2) dS = \int_S \rho x^2 dS + \int_S \rho y^2 dS = I_x + I_y$$but I don't think it will help you here, since the computations will be exactly the same, just in two parts (so in fact it is more long-winded).

The double integral is not too bad... I promise! For starters, can you evaluate$$\int_{-\frac{x}{\tan{\alpha}}}^{\frac{x}{\tan{\alpha}}} \rho (x^2 + y^2) dy$$whilst treating ##x## as a constant?
 
  • #8
Nexus99 said:
Homework Statement:: Find the moment of inertia of an isosceles triangle of mass M = 1.0 kg, height h = 0.4 m and base angles equal to ## \alpha = \frac{ \pi}{6} ##, with respect to an axis passing through its vertex
Relevant Equations:: moment of inertia

View attachment 268183
I did in this way:
## I = \int dm \rho^2 ##
Dividing the triangle in small rectangles with ##dA = dy x(y) ## where ##x(y) = 2 ctg( \alpha ) (h - y) ##
we have : ## dm = \sigma 2 ctg( \alpha ) (h - y) ##
Now i have ## \rho^2 = x^2 + (h-y)^2 ##
Now I don't know what I can do because it would be an integral in 2 variables that I don't know how to do
I like to think of this sort of thing visually before starting down the road with formulas.

When you do a double integral, you are usually laying things out in rows and columns. You integrate down a row to get the moment of inertia of a single row. Then you add those moments up -- integrating up in the column direction.

The first challenge you always face is trying to figure out an orientation for your rows and columns. There are many possibilities here:

1. You could consider a bunch of parallel strips running perpendicular to the base, up in the general direction of the apex, find the moments of inertia of each of those and then integrate them up.

2. You could consider a bunch of narrow triangles running from apex down toward the base, find the moments of inertia of each of those and then integrate those up.

3. You could consider a bunch of strips parallel to the base, short rows near the apex and long rows near the base. This is where @etotheipi has taken you.

The moment of inertia of a thin strip about its midpoint is a well known formula. That plus the parallel axis theorem gets you to the moment of inertia of a thin strip about the apex here.
 
Last edited:
  • Like
Likes Lnewqban and etotheipi
  • #9
@jbriggs444 @etotheipi
thanks for explanation.
I watched some videos on youtube about double integrals and i got these results
## I_z = \frac{5 \rho h^4}{12 tan( \alpha) } = \frac{5 M h^2}{12} ##
Hope it's correct
 
  • #10
I noticed in this moment that is present the solution of this problem:

"For the calculation of the moment of inertia, can be considered the moment of inertia of the elementary rod with mass dm, thickness dy and length l = l (y) indicated in the figure, where y is the distance from the apex of the triangle . In this case, for Steiner's theorem the moment of inertia can be written as:"

Cattura.PNG

Cattura.PNG

Sorry if the solution is not written in english, but there was written nothing important, so I simply tried to explain what was written with arrows
 
  • #11
Nexus99 said:
... with respect to an axis passing through its vertex
...

It seems that the axis of rotation is parallel to the plane of the triangle, according to angle θ shown in the schematic; could it be perpendicular instead?

https://en.m.wikipedia.org/wiki/Centroid#Of_a_triangle

"The centroid is also the physical center of mass if the triangle is made from a uniform sheet of material; or if all the mass is concentrated at the three vertices, and evenly divided among them."

http://hyperphysics.phy-astr.gsu.edu/hbase/parax.html#pax

"The moment of inertia about a parallel axis is the center of mass moment plus the moment of inertia of the entire object treated as a point mass at the center of mass."
 
Last edited:
  • #12
Lnewqban said:
Could it be perpendicular instead?
It isAnyway i don't understand what do you want to say with your previous message
 
Last edited:
  • #13
Nexus99 said:
It isAnyway i didn't understand what do you want to say with your previous message
If I understand correctly, the axis of rotation is perpendicular to the plane of the triangle.
In other words, the triangle rotates about the dot, clockwise or counterclockwise on the plane over which it is represented.

Sorry about my previous post and links, perhaps I am over-simplifying the problem when I see it as estimating the moment of inertia (I) of a center of mass (m) located at the centroid of the triangle and rotating perpendicularly at a fixed distance (r) from an axis.
 
  • #14
Lnewqban said:
If I understand correctly, the axis of rotation is perpendicular to the plane of the triangle.
In other words, the triangle rotates about the dot, clockwise or counterclockwise on the plane over which it is represented.
Yes, it's a physical pendulum.
Sorry if it was not so clear but i had to summarise the problem
 

1. What is the formula for calculating the moment of inertia of an isosceles triangle?

The formula for calculating the moment of inertia of an isosceles triangle is I = (1/12) * b * h^3, where b is the base of the triangle and h is the height.

2. How is the moment of inertia different for an isosceles triangle compared to other shapes?

The moment of inertia for an isosceles triangle is different because it takes into account the unequal distribution of mass in the shape. The two equal sides of the triangle have a greater concentration of mass compared to the base, resulting in a different moment of inertia calculation.

3. Why is the moment of inertia important in physics?

The moment of inertia is important in physics because it helps determine how an object will rotate around a fixed axis. It is also used in calculating the angular momentum of an object and understanding its stability and motion.

4. Can the moment of inertia of an isosceles triangle be negative?

No, the moment of inertia of an isosceles triangle cannot be negative. It is always a positive value as it represents the resistance of the object to changes in its rotational motion.

5. How does the moment of inertia change if the isosceles triangle is rotated around a different axis?

The moment of inertia will change if the isosceles triangle is rotated around a different axis. It will be different for each axis, depending on the distance of the axis from the center of mass of the triangle. The further the axis is from the center of mass, the higher the moment of inertia will be.

Similar threads

  • Introductory Physics Homework Help
Replies
8
Views
1K
  • Introductory Physics Homework Help
2
Replies
52
Views
2K
Replies
25
Views
464
  • Introductory Physics Homework Help
Replies
13
Views
1K
  • Introductory Physics Homework Help
Replies
5
Views
1K
Replies
17
Views
1K
  • Introductory Physics Homework Help
Replies
3
Views
1K
  • Introductory Physics Homework Help
Replies
2
Views
635
  • Introductory Physics Homework Help
Replies
2
Views
159
  • Introductory Physics Homework Help
Replies
3
Views
816
Back
Top